Difference between revisions of "2021 AMC 12B Problems/Problem 1"

(Solution 2)
Line 12: Line 12:
 
== Video Solution by OmegaLearn (System of Equations) ==
 
== Video Solution by OmegaLearn (System of Equations) ==
 
https://youtu.be/hyYg62tT0sY
 
https://youtu.be/hyYg62tT0sY
 +
 +
==See Also==
 +
{{AMC12 box|year=2021|ab=B|before=First Problem|num-a=2}}
 +
{{MAA Notice}}

Revision as of 21:09, 11 February 2021

How many integer values of $x$ satisfy $|x|<3\pi$?

$\textbf{(A)} ~9 \qquad\textbf{(B)} ~10 \qquad\textbf{(C)} ~18 \qquad\textbf{(D)} ~19 \qquad\textbf{(E)} ~20$

Solution

Since $3\pi$ is about $9.42$, we multiply 9 by 2 and add 1 to get $\boxed{\textbf{(D)}\ ~19}$~smarty101

Solution 2

$|x|<3\pi$ $\iff$ $-3\pi<x<3\pi$. Since $\pi$ is approximately $3.14$, $3\pi$ is approximately $9.42$. We are trying to solve for $-9.42<x<9.42$, where $x\in\mathbb{Z}$. Hence, $-9.42<x<9.42$ $\implies$ $-9\leq x\leq9$, for $x\in\mathbb{Z}$. The number of integer values of $x$ is $9-(-9)+1=19$. Therefore, the answer is $\boxed{\textbf{(D)}19}$.

~ {TSun} ~

Video Solution by OmegaLearn (System of Equations)

https://youtu.be/hyYg62tT0sY

See Also

2021 AMC 12B (ProblemsAnswer KeyResources)
Preceded by
First Problem
Followed by
Problem 2
1 2 3 4 5 6 7 8 9 10 11 12 13 14 15 16 17 18 19 20 21 22 23 24 25
All AMC 12 Problems and Solutions

The problems on this page are copyrighted by the Mathematical Association of America's American Mathematics Competitions. AMC logo.png